Proof of a Stronger Version of Dirichlet's Approximation Theorem












1












$begingroup$


If $alpha$ is a real number and $n$ is a positive integer, there are integers $a$ and $b$ such that $1leq a leq n$ and $|alpha a - b| < frac{1}{n+1}$.



Here is an attempt of the proof. I'm more or less skeptical of the second case and on whether or not the cases are exhaustive. Ultimately, I am wondering if the proof is alright.



Notation



Let $x in mathbb{Z}$. Then I use $[x]$ to denote the floor of $x$.



A real number $rho$ can be expressed as the sum of its integer $[rho]$ and fractional parts ${rho}$ as $rho = [rho] + {rho}$ where $0 leq {rho} < 1$.



Proof



Consider the $n+2$ fractional parts ${jalpha}$ where $j = 0,cdots,n+1$.



Then partition the interval $[0,1)$ into the $n+1$ subintervals



$$Big[frac{k-1}{n+1},frac{k}{n+1}Big)$$



Then each fractional part lies in one of the subintervals.



Since there are $n+2$ numbers but only $n+1$ intervals, the Pigeonhole principle says that at least two of these numbers lie in the same interval. Further, because each interval has length $frac{1}{n+1}$ and does not include its right endpoint, each point in a given interval will be less than $frac{1}{n+1}$.



Hence, there are integers $0 leq p < q leq n+1$ such that $|{qalpha} - {palpha}| < frac{1}{n+1}$.



Upon manipulating the inequality of $p$ and $q$ we get



$$0 leq p leq q-1 leq n$$



First, assume that $p< q-1$. Then $1 leq q-1-pleq n$.



Therefore, let $a = q-1-p$ and $b = [qalpha]-[palpha]-alpha$.



Now, we have



$$begin{align}|aalpha-b| &= |(q-1-p)alpha - ([qalpha]-[palpha]-alpha)| \\ &= |(alpha q - [alpha q]) - (alpha p - [alpha p])| \\ &= |{alpha q} - {alpha p}| \\ &< frac{1}{n+1}end{align}$$



So the given choices for $a$ and $b$ suffice.



Now suppose that $q-1 = p$. Then $1 = q-p$, which satisfies the inequality $1 leq q-p leq n$. Hence, let $a = q-p$ and $b=[alpha q] - [alpha p]$. Then



$$begin{align}|a alpha - b| &= |(q-p)alpha - ([alpha q] - [alpha p])| \\ &= |(alpha q-[alpha q]) - (alpha p - [alpha p])| \\ &= |{alpha q} - {alpha p}| \\ &< frac{1}{n+1}end{align}$$



Thus the given choies for $a$ and $b$ suffice.



Upon exhausting all possibilities, we conclude that one of first $n$ multiples of a real number $alpha$ must be within $frac{1}{n+1}$ of an integer.










share|cite|improve this question











$endgroup$












  • $begingroup$
    There is a flaw: you allow j to range from 0 to n+1. However, when j = n+1, it isn't in the open interval [0,1), so the pigeonhole principle doesn't apply. One way you could fix this proof is to instead have numbers {ja} for j = 0..n, and 1. Now you have two cases: when you have {pa} and {qa} in the same interval, or when you have 1 and {pa} in the last interval. Then you can solve for that, using the definition of the fractional part.
    $endgroup$
    – Peter Wang
    Jan 29 '18 at 21:57










  • $begingroup$
    this answer is related.
    $endgroup$
    – robjohn
    Jan 29 '18 at 22:57
















1












$begingroup$


If $alpha$ is a real number and $n$ is a positive integer, there are integers $a$ and $b$ such that $1leq a leq n$ and $|alpha a - b| < frac{1}{n+1}$.



Here is an attempt of the proof. I'm more or less skeptical of the second case and on whether or not the cases are exhaustive. Ultimately, I am wondering if the proof is alright.



Notation



Let $x in mathbb{Z}$. Then I use $[x]$ to denote the floor of $x$.



A real number $rho$ can be expressed as the sum of its integer $[rho]$ and fractional parts ${rho}$ as $rho = [rho] + {rho}$ where $0 leq {rho} < 1$.



Proof



Consider the $n+2$ fractional parts ${jalpha}$ where $j = 0,cdots,n+1$.



Then partition the interval $[0,1)$ into the $n+1$ subintervals



$$Big[frac{k-1}{n+1},frac{k}{n+1}Big)$$



Then each fractional part lies in one of the subintervals.



Since there are $n+2$ numbers but only $n+1$ intervals, the Pigeonhole principle says that at least two of these numbers lie in the same interval. Further, because each interval has length $frac{1}{n+1}$ and does not include its right endpoint, each point in a given interval will be less than $frac{1}{n+1}$.



Hence, there are integers $0 leq p < q leq n+1$ such that $|{qalpha} - {palpha}| < frac{1}{n+1}$.



Upon manipulating the inequality of $p$ and $q$ we get



$$0 leq p leq q-1 leq n$$



First, assume that $p< q-1$. Then $1 leq q-1-pleq n$.



Therefore, let $a = q-1-p$ and $b = [qalpha]-[palpha]-alpha$.



Now, we have



$$begin{align}|aalpha-b| &= |(q-1-p)alpha - ([qalpha]-[palpha]-alpha)| \\ &= |(alpha q - [alpha q]) - (alpha p - [alpha p])| \\ &= |{alpha q} - {alpha p}| \\ &< frac{1}{n+1}end{align}$$



So the given choices for $a$ and $b$ suffice.



Now suppose that $q-1 = p$. Then $1 = q-p$, which satisfies the inequality $1 leq q-p leq n$. Hence, let $a = q-p$ and $b=[alpha q] - [alpha p]$. Then



$$begin{align}|a alpha - b| &= |(q-p)alpha - ([alpha q] - [alpha p])| \\ &= |(alpha q-[alpha q]) - (alpha p - [alpha p])| \\ &= |{alpha q} - {alpha p}| \\ &< frac{1}{n+1}end{align}$$



Thus the given choies for $a$ and $b$ suffice.



Upon exhausting all possibilities, we conclude that one of first $n$ multiples of a real number $alpha$ must be within $frac{1}{n+1}$ of an integer.










share|cite|improve this question











$endgroup$












  • $begingroup$
    There is a flaw: you allow j to range from 0 to n+1. However, when j = n+1, it isn't in the open interval [0,1), so the pigeonhole principle doesn't apply. One way you could fix this proof is to instead have numbers {ja} for j = 0..n, and 1. Now you have two cases: when you have {pa} and {qa} in the same interval, or when you have 1 and {pa} in the last interval. Then you can solve for that, using the definition of the fractional part.
    $endgroup$
    – Peter Wang
    Jan 29 '18 at 21:57










  • $begingroup$
    this answer is related.
    $endgroup$
    – robjohn
    Jan 29 '18 at 22:57














1












1








1


1



$begingroup$


If $alpha$ is a real number and $n$ is a positive integer, there are integers $a$ and $b$ such that $1leq a leq n$ and $|alpha a - b| < frac{1}{n+1}$.



Here is an attempt of the proof. I'm more or less skeptical of the second case and on whether or not the cases are exhaustive. Ultimately, I am wondering if the proof is alright.



Notation



Let $x in mathbb{Z}$. Then I use $[x]$ to denote the floor of $x$.



A real number $rho$ can be expressed as the sum of its integer $[rho]$ and fractional parts ${rho}$ as $rho = [rho] + {rho}$ where $0 leq {rho} < 1$.



Proof



Consider the $n+2$ fractional parts ${jalpha}$ where $j = 0,cdots,n+1$.



Then partition the interval $[0,1)$ into the $n+1$ subintervals



$$Big[frac{k-1}{n+1},frac{k}{n+1}Big)$$



Then each fractional part lies in one of the subintervals.



Since there are $n+2$ numbers but only $n+1$ intervals, the Pigeonhole principle says that at least two of these numbers lie in the same interval. Further, because each interval has length $frac{1}{n+1}$ and does not include its right endpoint, each point in a given interval will be less than $frac{1}{n+1}$.



Hence, there are integers $0 leq p < q leq n+1$ such that $|{qalpha} - {palpha}| < frac{1}{n+1}$.



Upon manipulating the inequality of $p$ and $q$ we get



$$0 leq p leq q-1 leq n$$



First, assume that $p< q-1$. Then $1 leq q-1-pleq n$.



Therefore, let $a = q-1-p$ and $b = [qalpha]-[palpha]-alpha$.



Now, we have



$$begin{align}|aalpha-b| &= |(q-1-p)alpha - ([qalpha]-[palpha]-alpha)| \\ &= |(alpha q - [alpha q]) - (alpha p - [alpha p])| \\ &= |{alpha q} - {alpha p}| \\ &< frac{1}{n+1}end{align}$$



So the given choices for $a$ and $b$ suffice.



Now suppose that $q-1 = p$. Then $1 = q-p$, which satisfies the inequality $1 leq q-p leq n$. Hence, let $a = q-p$ and $b=[alpha q] - [alpha p]$. Then



$$begin{align}|a alpha - b| &= |(q-p)alpha - ([alpha q] - [alpha p])| \\ &= |(alpha q-[alpha q]) - (alpha p - [alpha p])| \\ &= |{alpha q} - {alpha p}| \\ &< frac{1}{n+1}end{align}$$



Thus the given choies for $a$ and $b$ suffice.



Upon exhausting all possibilities, we conclude that one of first $n$ multiples of a real number $alpha$ must be within $frac{1}{n+1}$ of an integer.










share|cite|improve this question











$endgroup$




If $alpha$ is a real number and $n$ is a positive integer, there are integers $a$ and $b$ such that $1leq a leq n$ and $|alpha a - b| < frac{1}{n+1}$.



Here is an attempt of the proof. I'm more or less skeptical of the second case and on whether or not the cases are exhaustive. Ultimately, I am wondering if the proof is alright.



Notation



Let $x in mathbb{Z}$. Then I use $[x]$ to denote the floor of $x$.



A real number $rho$ can be expressed as the sum of its integer $[rho]$ and fractional parts ${rho}$ as $rho = [rho] + {rho}$ where $0 leq {rho} < 1$.



Proof



Consider the $n+2$ fractional parts ${jalpha}$ where $j = 0,cdots,n+1$.



Then partition the interval $[0,1)$ into the $n+1$ subintervals



$$Big[frac{k-1}{n+1},frac{k}{n+1}Big)$$



Then each fractional part lies in one of the subintervals.



Since there are $n+2$ numbers but only $n+1$ intervals, the Pigeonhole principle says that at least two of these numbers lie in the same interval. Further, because each interval has length $frac{1}{n+1}$ and does not include its right endpoint, each point in a given interval will be less than $frac{1}{n+1}$.



Hence, there are integers $0 leq p < q leq n+1$ such that $|{qalpha} - {palpha}| < frac{1}{n+1}$.



Upon manipulating the inequality of $p$ and $q$ we get



$$0 leq p leq q-1 leq n$$



First, assume that $p< q-1$. Then $1 leq q-1-pleq n$.



Therefore, let $a = q-1-p$ and $b = [qalpha]-[palpha]-alpha$.



Now, we have



$$begin{align}|aalpha-b| &= |(q-1-p)alpha - ([qalpha]-[palpha]-alpha)| \\ &= |(alpha q - [alpha q]) - (alpha p - [alpha p])| \\ &= |{alpha q} - {alpha p}| \\ &< frac{1}{n+1}end{align}$$



So the given choices for $a$ and $b$ suffice.



Now suppose that $q-1 = p$. Then $1 = q-p$, which satisfies the inequality $1 leq q-p leq n$. Hence, let $a = q-p$ and $b=[alpha q] - [alpha p]$. Then



$$begin{align}|a alpha - b| &= |(q-p)alpha - ([alpha q] - [alpha p])| \\ &= |(alpha q-[alpha q]) - (alpha p - [alpha p])| \\ &= |{alpha q} - {alpha p}| \\ &< frac{1}{n+1}end{align}$$



Thus the given choies for $a$ and $b$ suffice.



Upon exhausting all possibilities, we conclude that one of first $n$ multiples of a real number $alpha$ must be within $frac{1}{n+1}$ of an integer.







elementary-number-theory proof-verification






share|cite|improve this question















share|cite|improve this question













share|cite|improve this question




share|cite|improve this question








edited Jun 1 '16 at 0:29







Benedict Voltaire

















asked May 28 '16 at 23:35









Benedict VoltaireBenedict Voltaire

1,299928




1,299928












  • $begingroup$
    There is a flaw: you allow j to range from 0 to n+1. However, when j = n+1, it isn't in the open interval [0,1), so the pigeonhole principle doesn't apply. One way you could fix this proof is to instead have numbers {ja} for j = 0..n, and 1. Now you have two cases: when you have {pa} and {qa} in the same interval, or when you have 1 and {pa} in the last interval. Then you can solve for that, using the definition of the fractional part.
    $endgroup$
    – Peter Wang
    Jan 29 '18 at 21:57










  • $begingroup$
    this answer is related.
    $endgroup$
    – robjohn
    Jan 29 '18 at 22:57


















  • $begingroup$
    There is a flaw: you allow j to range from 0 to n+1. However, when j = n+1, it isn't in the open interval [0,1), so the pigeonhole principle doesn't apply. One way you could fix this proof is to instead have numbers {ja} for j = 0..n, and 1. Now you have two cases: when you have {pa} and {qa} in the same interval, or when you have 1 and {pa} in the last interval. Then you can solve for that, using the definition of the fractional part.
    $endgroup$
    – Peter Wang
    Jan 29 '18 at 21:57










  • $begingroup$
    this answer is related.
    $endgroup$
    – robjohn
    Jan 29 '18 at 22:57
















$begingroup$
There is a flaw: you allow j to range from 0 to n+1. However, when j = n+1, it isn't in the open interval [0,1), so the pigeonhole principle doesn't apply. One way you could fix this proof is to instead have numbers {ja} for j = 0..n, and 1. Now you have two cases: when you have {pa} and {qa} in the same interval, or when you have 1 and {pa} in the last interval. Then you can solve for that, using the definition of the fractional part.
$endgroup$
– Peter Wang
Jan 29 '18 at 21:57




$begingroup$
There is a flaw: you allow j to range from 0 to n+1. However, when j = n+1, it isn't in the open interval [0,1), so the pigeonhole principle doesn't apply. One way you could fix this proof is to instead have numbers {ja} for j = 0..n, and 1. Now you have two cases: when you have {pa} and {qa} in the same interval, or when you have 1 and {pa} in the last interval. Then you can solve for that, using the definition of the fractional part.
$endgroup$
– Peter Wang
Jan 29 '18 at 21:57












$begingroup$
this answer is related.
$endgroup$
– robjohn
Jan 29 '18 at 22:57




$begingroup$
this answer is related.
$endgroup$
– robjohn
Jan 29 '18 at 22:57










2 Answers
2






active

oldest

votes


















1












$begingroup$

My comment is wrong: the problem in your proof is that $b$ must be an integer, and in your first case, where $p < q-1$, you have $b$ to be $([qalpha] - [palpha] - alpha)$, but since $alpha$ is a real number, b is not necessarily an integer.



Here is how I did the proof (it's really similar and only differs in one small aspect):



Consider the $n+2$ numbers: $0, {alpha}, {2alpha}, ..., {nalpha}, 1$.



Now divide the closed interval $[0,1]$ into $n+1$ partitions:



$[0, frac{1}{n+1}), [frac{1}{n+1}, frac{2}{n+1}), ..., [frac{n}{n+1}, 1]$



By the pigeonhole principle, since all $n+2$ numbers fall in the interval $[0,1]$, there must be one of the $n+1$ subintervals that contains two numbers. Now, there are two cases:





Case 1: The two numbers are in the first $n$ subintervals, subinterval $i$, $[frac{i-1}{n+1},frac{i}{n+1})$.



Let $p, q in mathbb{Z}$ such that $0leq p lt q leq n$ without loss of generality. We have $|{qalpha} - {palpha}| lt frac{1}{n+1}$



Thus, by definition of ${}$, we have $|qalpha - lfloor qalpha rfloor - (palpha - lfloor palpha rfloor)| lt frac{1}{n+1}$



Rearranging, we have $|(q-p)alpha - (lfloor qalpha rfloor - lfloor palpha rfloor)| lt frac{1}{n+1}$



Take $a = q-p$ and $b = lfloor qalpha rfloor - lfloor palpha rfloor$, both of which are integers, to satisfy the inequality.



Case 2: The two numbers are in the last subinterval, $[frac{n}{n+1}, 1]$. One of the numbers must be 1. Let the other number be ${palpha}$, for $p in [0,n]$.



Thus, we have $|{palpha} - 1| leq frac{1}{n+1}$. Note that it is less than or equal to because this is the only closed interval.



Simplifying as we did above, we get $|palpha - (lfloor palpha rfloor + 1)| leq frac{1}{n+1}$.



Take $a = p$ and $b = lfloor palpha rfloor + 1$, satisfying the constraints. QED.






share|cite|improve this answer









$endgroup$





















    0












    $begingroup$

    Your proof is wrong particularly in the first case:




    First, assume that $p<q−1$. Then $1 le q−1−p le n$.




    This is clearly wrong, take $p=1$, $q=2$. Then, $q-1-p=0 < 1$.



    You can intuitively understand why the proof is wrong by noticing that you can have $a>n$ when $p=0$ and $q=n+1$, which would invalidate the entire proof.






    share|cite|improve this answer









    $endgroup$













      Your Answer





      StackExchange.ifUsing("editor", function () {
      return StackExchange.using("mathjaxEditing", function () {
      StackExchange.MarkdownEditor.creationCallbacks.add(function (editor, postfix) {
      StackExchange.mathjaxEditing.prepareWmdForMathJax(editor, postfix, [["$", "$"], ["\\(","\\)"]]);
      });
      });
      }, "mathjax-editing");

      StackExchange.ready(function() {
      var channelOptions = {
      tags: "".split(" "),
      id: "69"
      };
      initTagRenderer("".split(" "), "".split(" "), channelOptions);

      StackExchange.using("externalEditor", function() {
      // Have to fire editor after snippets, if snippets enabled
      if (StackExchange.settings.snippets.snippetsEnabled) {
      StackExchange.using("snippets", function() {
      createEditor();
      });
      }
      else {
      createEditor();
      }
      });

      function createEditor() {
      StackExchange.prepareEditor({
      heartbeatType: 'answer',
      autoActivateHeartbeat: false,
      convertImagesToLinks: true,
      noModals: true,
      showLowRepImageUploadWarning: true,
      reputationToPostImages: 10,
      bindNavPrevention: true,
      postfix: "",
      imageUploader: {
      brandingHtml: "Powered by u003ca class="icon-imgur-white" href="https://imgur.com/"u003eu003c/au003e",
      contentPolicyHtml: "User contributions licensed under u003ca href="https://creativecommons.org/licenses/by-sa/3.0/"u003ecc by-sa 3.0 with attribution requiredu003c/au003e u003ca href="https://stackoverflow.com/legal/content-policy"u003e(content policy)u003c/au003e",
      allowUrls: true
      },
      noCode: true, onDemand: true,
      discardSelector: ".discard-answer"
      ,immediatelyShowMarkdownHelp:true
      });


      }
      });














      draft saved

      draft discarded


















      StackExchange.ready(
      function () {
      StackExchange.openid.initPostLogin('.new-post-login', 'https%3a%2f%2fmath.stackexchange.com%2fquestions%2f1803988%2fproof-of-a-stronger-version-of-dirichlets-approximation-theorem%23new-answer', 'question_page');
      }
      );

      Post as a guest















      Required, but never shown

























      2 Answers
      2






      active

      oldest

      votes








      2 Answers
      2






      active

      oldest

      votes









      active

      oldest

      votes






      active

      oldest

      votes









      1












      $begingroup$

      My comment is wrong: the problem in your proof is that $b$ must be an integer, and in your first case, where $p < q-1$, you have $b$ to be $([qalpha] - [palpha] - alpha)$, but since $alpha$ is a real number, b is not necessarily an integer.



      Here is how I did the proof (it's really similar and only differs in one small aspect):



      Consider the $n+2$ numbers: $0, {alpha}, {2alpha}, ..., {nalpha}, 1$.



      Now divide the closed interval $[0,1]$ into $n+1$ partitions:



      $[0, frac{1}{n+1}), [frac{1}{n+1}, frac{2}{n+1}), ..., [frac{n}{n+1}, 1]$



      By the pigeonhole principle, since all $n+2$ numbers fall in the interval $[0,1]$, there must be one of the $n+1$ subintervals that contains two numbers. Now, there are two cases:





      Case 1: The two numbers are in the first $n$ subintervals, subinterval $i$, $[frac{i-1}{n+1},frac{i}{n+1})$.



      Let $p, q in mathbb{Z}$ such that $0leq p lt q leq n$ without loss of generality. We have $|{qalpha} - {palpha}| lt frac{1}{n+1}$



      Thus, by definition of ${}$, we have $|qalpha - lfloor qalpha rfloor - (palpha - lfloor palpha rfloor)| lt frac{1}{n+1}$



      Rearranging, we have $|(q-p)alpha - (lfloor qalpha rfloor - lfloor palpha rfloor)| lt frac{1}{n+1}$



      Take $a = q-p$ and $b = lfloor qalpha rfloor - lfloor palpha rfloor$, both of which are integers, to satisfy the inequality.



      Case 2: The two numbers are in the last subinterval, $[frac{n}{n+1}, 1]$. One of the numbers must be 1. Let the other number be ${palpha}$, for $p in [0,n]$.



      Thus, we have $|{palpha} - 1| leq frac{1}{n+1}$. Note that it is less than or equal to because this is the only closed interval.



      Simplifying as we did above, we get $|palpha - (lfloor palpha rfloor + 1)| leq frac{1}{n+1}$.



      Take $a = p$ and $b = lfloor palpha rfloor + 1$, satisfying the constraints. QED.






      share|cite|improve this answer









      $endgroup$


















        1












        $begingroup$

        My comment is wrong: the problem in your proof is that $b$ must be an integer, and in your first case, where $p < q-1$, you have $b$ to be $([qalpha] - [palpha] - alpha)$, but since $alpha$ is a real number, b is not necessarily an integer.



        Here is how I did the proof (it's really similar and only differs in one small aspect):



        Consider the $n+2$ numbers: $0, {alpha}, {2alpha}, ..., {nalpha}, 1$.



        Now divide the closed interval $[0,1]$ into $n+1$ partitions:



        $[0, frac{1}{n+1}), [frac{1}{n+1}, frac{2}{n+1}), ..., [frac{n}{n+1}, 1]$



        By the pigeonhole principle, since all $n+2$ numbers fall in the interval $[0,1]$, there must be one of the $n+1$ subintervals that contains two numbers. Now, there are two cases:





        Case 1: The two numbers are in the first $n$ subintervals, subinterval $i$, $[frac{i-1}{n+1},frac{i}{n+1})$.



        Let $p, q in mathbb{Z}$ such that $0leq p lt q leq n$ without loss of generality. We have $|{qalpha} - {palpha}| lt frac{1}{n+1}$



        Thus, by definition of ${}$, we have $|qalpha - lfloor qalpha rfloor - (palpha - lfloor palpha rfloor)| lt frac{1}{n+1}$



        Rearranging, we have $|(q-p)alpha - (lfloor qalpha rfloor - lfloor palpha rfloor)| lt frac{1}{n+1}$



        Take $a = q-p$ and $b = lfloor qalpha rfloor - lfloor palpha rfloor$, both of which are integers, to satisfy the inequality.



        Case 2: The two numbers are in the last subinterval, $[frac{n}{n+1}, 1]$. One of the numbers must be 1. Let the other number be ${palpha}$, for $p in [0,n]$.



        Thus, we have $|{palpha} - 1| leq frac{1}{n+1}$. Note that it is less than or equal to because this is the only closed interval.



        Simplifying as we did above, we get $|palpha - (lfloor palpha rfloor + 1)| leq frac{1}{n+1}$.



        Take $a = p$ and $b = lfloor palpha rfloor + 1$, satisfying the constraints. QED.






        share|cite|improve this answer









        $endgroup$
















          1












          1








          1





          $begingroup$

          My comment is wrong: the problem in your proof is that $b$ must be an integer, and in your first case, where $p < q-1$, you have $b$ to be $([qalpha] - [palpha] - alpha)$, but since $alpha$ is a real number, b is not necessarily an integer.



          Here is how I did the proof (it's really similar and only differs in one small aspect):



          Consider the $n+2$ numbers: $0, {alpha}, {2alpha}, ..., {nalpha}, 1$.



          Now divide the closed interval $[0,1]$ into $n+1$ partitions:



          $[0, frac{1}{n+1}), [frac{1}{n+1}, frac{2}{n+1}), ..., [frac{n}{n+1}, 1]$



          By the pigeonhole principle, since all $n+2$ numbers fall in the interval $[0,1]$, there must be one of the $n+1$ subintervals that contains two numbers. Now, there are two cases:





          Case 1: The two numbers are in the first $n$ subintervals, subinterval $i$, $[frac{i-1}{n+1},frac{i}{n+1})$.



          Let $p, q in mathbb{Z}$ such that $0leq p lt q leq n$ without loss of generality. We have $|{qalpha} - {palpha}| lt frac{1}{n+1}$



          Thus, by definition of ${}$, we have $|qalpha - lfloor qalpha rfloor - (palpha - lfloor palpha rfloor)| lt frac{1}{n+1}$



          Rearranging, we have $|(q-p)alpha - (lfloor qalpha rfloor - lfloor palpha rfloor)| lt frac{1}{n+1}$



          Take $a = q-p$ and $b = lfloor qalpha rfloor - lfloor palpha rfloor$, both of which are integers, to satisfy the inequality.



          Case 2: The two numbers are in the last subinterval, $[frac{n}{n+1}, 1]$. One of the numbers must be 1. Let the other number be ${palpha}$, for $p in [0,n]$.



          Thus, we have $|{palpha} - 1| leq frac{1}{n+1}$. Note that it is less than or equal to because this is the only closed interval.



          Simplifying as we did above, we get $|palpha - (lfloor palpha rfloor + 1)| leq frac{1}{n+1}$.



          Take $a = p$ and $b = lfloor palpha rfloor + 1$, satisfying the constraints. QED.






          share|cite|improve this answer









          $endgroup$



          My comment is wrong: the problem in your proof is that $b$ must be an integer, and in your first case, where $p < q-1$, you have $b$ to be $([qalpha] - [palpha] - alpha)$, but since $alpha$ is a real number, b is not necessarily an integer.



          Here is how I did the proof (it's really similar and only differs in one small aspect):



          Consider the $n+2$ numbers: $0, {alpha}, {2alpha}, ..., {nalpha}, 1$.



          Now divide the closed interval $[0,1]$ into $n+1$ partitions:



          $[0, frac{1}{n+1}), [frac{1}{n+1}, frac{2}{n+1}), ..., [frac{n}{n+1}, 1]$



          By the pigeonhole principle, since all $n+2$ numbers fall in the interval $[0,1]$, there must be one of the $n+1$ subintervals that contains two numbers. Now, there are two cases:





          Case 1: The two numbers are in the first $n$ subintervals, subinterval $i$, $[frac{i-1}{n+1},frac{i}{n+1})$.



          Let $p, q in mathbb{Z}$ such that $0leq p lt q leq n$ without loss of generality. We have $|{qalpha} - {palpha}| lt frac{1}{n+1}$



          Thus, by definition of ${}$, we have $|qalpha - lfloor qalpha rfloor - (palpha - lfloor palpha rfloor)| lt frac{1}{n+1}$



          Rearranging, we have $|(q-p)alpha - (lfloor qalpha rfloor - lfloor palpha rfloor)| lt frac{1}{n+1}$



          Take $a = q-p$ and $b = lfloor qalpha rfloor - lfloor palpha rfloor$, both of which are integers, to satisfy the inequality.



          Case 2: The two numbers are in the last subinterval, $[frac{n}{n+1}, 1]$. One of the numbers must be 1. Let the other number be ${palpha}$, for $p in [0,n]$.



          Thus, we have $|{palpha} - 1| leq frac{1}{n+1}$. Note that it is less than or equal to because this is the only closed interval.



          Simplifying as we did above, we get $|palpha - (lfloor palpha rfloor + 1)| leq frac{1}{n+1}$.



          Take $a = p$ and $b = lfloor palpha rfloor + 1$, satisfying the constraints. QED.







          share|cite|improve this answer












          share|cite|improve this answer



          share|cite|improve this answer










          answered Jan 29 '18 at 22:23









          Peter WangPeter Wang

          1114




          1114























              0












              $begingroup$

              Your proof is wrong particularly in the first case:




              First, assume that $p<q−1$. Then $1 le q−1−p le n$.




              This is clearly wrong, take $p=1$, $q=2$. Then, $q-1-p=0 < 1$.



              You can intuitively understand why the proof is wrong by noticing that you can have $a>n$ when $p=0$ and $q=n+1$, which would invalidate the entire proof.






              share|cite|improve this answer









              $endgroup$


















                0












                $begingroup$

                Your proof is wrong particularly in the first case:




                First, assume that $p<q−1$. Then $1 le q−1−p le n$.




                This is clearly wrong, take $p=1$, $q=2$. Then, $q-1-p=0 < 1$.



                You can intuitively understand why the proof is wrong by noticing that you can have $a>n$ when $p=0$ and $q=n+1$, which would invalidate the entire proof.






                share|cite|improve this answer









                $endgroup$
















                  0












                  0








                  0





                  $begingroup$

                  Your proof is wrong particularly in the first case:




                  First, assume that $p<q−1$. Then $1 le q−1−p le n$.




                  This is clearly wrong, take $p=1$, $q=2$. Then, $q-1-p=0 < 1$.



                  You can intuitively understand why the proof is wrong by noticing that you can have $a>n$ when $p=0$ and $q=n+1$, which would invalidate the entire proof.






                  share|cite|improve this answer









                  $endgroup$



                  Your proof is wrong particularly in the first case:




                  First, assume that $p<q−1$. Then $1 le q−1−p le n$.




                  This is clearly wrong, take $p=1$, $q=2$. Then, $q-1-p=0 < 1$.



                  You can intuitively understand why the proof is wrong by noticing that you can have $a>n$ when $p=0$ and $q=n+1$, which would invalidate the entire proof.







                  share|cite|improve this answer












                  share|cite|improve this answer



                  share|cite|improve this answer










                  answered Jan 10 at 16:54









                  Jeb_is_a_messJeb_is_a_mess

                  306




                  306






























                      draft saved

                      draft discarded




















































                      Thanks for contributing an answer to Mathematics Stack Exchange!


                      • Please be sure to answer the question. Provide details and share your research!

                      But avoid



                      • Asking for help, clarification, or responding to other answers.

                      • Making statements based on opinion; back them up with references or personal experience.


                      Use MathJax to format equations. MathJax reference.


                      To learn more, see our tips on writing great answers.




                      draft saved


                      draft discarded














                      StackExchange.ready(
                      function () {
                      StackExchange.openid.initPostLogin('.new-post-login', 'https%3a%2f%2fmath.stackexchange.com%2fquestions%2f1803988%2fproof-of-a-stronger-version-of-dirichlets-approximation-theorem%23new-answer', 'question_page');
                      }
                      );

                      Post as a guest















                      Required, but never shown





















































                      Required, but never shown














                      Required, but never shown












                      Required, but never shown







                      Required, but never shown

































                      Required, but never shown














                      Required, but never shown












                      Required, but never shown







                      Required, but never shown







                      Popular posts from this blog

                      Mario Kart Wii

                      What does “Dominus providebit” mean?

                      Antonio Litta Visconti Arese